LSAT and Law School Admissions Forum

Get expert LSAT preparation and law school admissions advice from PowerScore Test Preparation.

 Administrator
PowerScore Staff
  • PowerScore Staff
  • Posts: 8919
  • Joined: Feb 02, 2011
|
#23065
Complete Question Explanation

Assumption. The correct answer choice is (B)

The school superintendent claims that the best high school was previously limited to children of those wealthy enough to pay the tuition. The superintendent announces that since now the only requirement is residence in the school's neighborhood, people who could not attend previously will now be able to attend.

The superintendent ignores the strong possibility that a school that serves the wealthy might just happen to be in a wealthy neighborhood. Since you are asked to identify the superintendent's necessary assumption, you should concentrate on that, although any number of other factors could also come into play.

Answer choice (A): The superintendent relies on the assumption that the residents of the neighborhood are not all wealthy, so this choice is somewhat contradictory and therefore incorrect.

Answer choice (B): This is the correct answer choice. The superintendent must assume that people other than those already wealthy enough to pay the tuition live in the school's neighborhood.

Answer choice (C): This choice is incorrect, because it goes too far. The superintendent only needs a few people who could not afford tuition to be enabled to attend, but this choice concerns whether most of the new students will be those whose parents could not previously pay.

Answer choice (D): The superintendent's argument concerned who goes to a specific high school, so information about whether students could attend another high school is irrelevant.

Answer choice (E): The superintendent argues that people will have the option, not that people will want to use it.
 reop6780
  • Posts: 265
  • Joined: Jul 27, 2013
|
#12692
(pg 135)

I understand that once the answer B is negated, the argument of school superintendent would be disproved; that is' answer B must be the correct answer.

However, I got this question wrong by selecting the answer E.
I thought the answer E must be the fundamental assumption that the conclusion relies on.
Once negated, it becomes a statement that "there are no people among those who could not afford the old tuition who wish to send their children to the school."
This definitely damages the argument in the stimuli in that he insists the new requirement would allow people who couldn't afford the tuition to send their kids to the school.

What I think is the problem of the E is that it is somehow stated in the stimuli, "Parents who were previously denied the option of sending their children to this option."

Since assumption is the unstated premise, answer E is wrong because it is already implied or stated in the stimuli....????

Are there many answer choices that are incorrect due to such matter ??
 Nikki Siclunov
PowerScore Staff
  • PowerScore Staff
  • Posts: 1362
  • Joined: Aug 02, 2011
|
#12726
Hey reop6780,

Thanks for your question!

The superintendent is arguing that by replacing the tuition requirement with a residency requirement, parents who were previously denied the option of sending their kids to school now have this option. And you are right: this line of reasoning relies on the fundamental assumption that there are at least some parents not wealthy enough to have paid the old tuition who wish to send their kids to the school (if only rich parents wanted to send their kids to the school, then the proposal would be meaningless). And indeed, we already know from the stimulus, because some parents were previously denied this option.

The problem with answer choice (E) lies in the word "many." We need not assume that many poor people would want to send their kids to the school: "some" would be enough. Your mistake was in forming the logical opposite: the logical opposite of "many people" is not "no people," but simply "not many people" (i.e. "few people"). So, even if only a few parents would want to take advantage of the new option, the conclusion still stands. After all, the author never argued that the school will become a bastion of economic equality :)

Hope this helps! Let me know :)
 reop6780
  • Posts: 265
  • Joined: Jul 27, 2013
|
#12742
Hi, Nikki!

First of all, thank you for the reply.

I just have one additional question found in your explanation.

When I studied the LR bible, the expression of "many" was categorized with "some."

Also, "at least one" is also bound with the same group with "some." (*It's on page 307 in LR bible)

Since every time I see "many," I automatically regard this as "some" or "at least one," I did not understand your distinction made between "many" and "some."

I do appreciate your confirmation that E is wrong based upon the fact it is the premise that is already stated in the stimuli.

I was just a little confused of your distinction made regarding "many."
 Nikki Siclunov
PowerScore Staff
  • PowerScore Staff
  • Posts: 1362
  • Joined: Aug 02, 2011
|
#12858
Hi reop6780,

The LSAT uses the common definition of words such as "some" and "many." Indeed, "some" means "at least one." The correct answer choices in quite a few Assumption and Must Be True questions contain this wording, because "some" is relatively easy to prove with the information contained in the stimulus. By contrast, "many" conveys multitude - not necessarily "most," but definitely "more than just a few." I would make a distinction between the following three expression groups:
  • At least one (i.e. 1 or more)
    Some
    Few
vs.
  • Many (definitely more than a few)
    A lot
    Quite a few
vs.
  • Most (i.e. 50% + 1)
    Majority of
Hope this helps!
 lorettan102
  • Posts: 11
  • Joined: Nov 02, 2016
|
#30482
I initially chose this answer because of the contenders of B, C and E, it makes the most sense. However, does the argument really rely on those not so wealthy being able to live in the neighborhood? Shouldn't the argument really rely more simply on the fact that those other than the wealthy people who are able to send their children to the school do live in the neighborhood?

Loretta
User avatar
 Jonathan Evans
PowerScore Staff
  • PowerScore Staff
  • Posts: 726
  • Joined: Jun 09, 2016
|
#30541
Loretta,

Good question. It does indeed rely on the assumption that at a minimum people must be able to live in the neighborhood of the school. In fact, it is not absolutely necessary that such people do live in this neighborhood.

The conclusion states that parents who previously were denied the option to send their children to this school will now have this option. Even if they do not currently live in the neighborhood of the school, they might have the option to send their children to this school if they are able to move into this neighborhood.

The key is the ability to live in the neighborhood of the school. Without this ability to live in the neighborhood of the school, there is absolutely no option for parents of the children in question to send their children to this school.

I hope this helps!
 na02
  • Posts: 31
  • Joined: Mar 19, 2019
|
#63603
I'm not entirely sure I see the difference between choices B and C...
I chose C because I got caught on "are able to live" for answer B; but both negated still undermine the superintendent's argument: B: "are not able to live" and C: "are not in the majority in the district"

Would you please explain the difference further?

Many thanks!
 Brook Miscoski
PowerScore Staff
  • PowerScore Staff
  • Posts: 418
  • Joined: Sep 13, 2018
|
#63894
na02,

The difference is mainly that (B) says that some students in the neighborhood cannot afford the school, while (C) says that the majority of the students in the neighborhood cannot afford the school.

Since you're doing an assumption question, you need to choose what's necessary to the argument. Majorities weren't at issue. Additionally, the negation test shows that (C) is wrong. When (B) is negated, the Superintendant's argument is wrong because there is no-one left who cannot afford the school. When (C) is negated, the Superintendant's argumennt survives because a minority of students still means some who cannot afford the school.

Get the most out of your LSAT Prep Plus subscription.

Analyze and track your performance with our Testing and Analytics Package.